Wiki Olimpédia
Advertisement

É interessante procurarmos, no problema, coisas que não variam.

Para que você consiga encontrar invariantes é útil entender bem as transformações pelo qual elas passam (ou as características principais dos elementos do problema). Por isso, é útil fazer vários exemplos para que você consiga encontrar um padrão.

Como Encontrar Invariantes[]

  • Faça pequenos casos
  • Estude como são as transformações e procure características importantes sobre elas
  • Em certos momentos, você deve fazer a própria invariante aparecer.

Exemplo (OBM 2012 - 3ª Fase - Nível 2)[]

Quando duas amebas vermelhas se juntam, se transformam em uma única ameba azul; quando uma ameba vermelha se junta com uma ameba azul, as duas se transformam em três amebas vermelhas; quando duas amebas azuis se juntam, elas se transformam em quatro amebas vermelhas. Um tubo de ensaio tem inicialmente amebas azuis e amebas vermelhas.

(a) É possível que após algumas transformações o tubo contenha amebas azuis e amebas vermelhas?

(b) É possível que após algumas transformações o tubo contenha amebas azuis e amebas vermelhas?

Solução:

(a) Vamos pensar nas transformações apenas em termos de cores (sem levar em conta as quantidades). Podemos transformar vermelhas em azuis (e vice-versa). Já se tivermos azuis e vermelhas juntas, podemos apenas transformá-las em vermelhas. Então, é mais "fácil" obtermos vermelhas do que azuis, nesse sentido.

Por isso, uma estratégia interessante separarmos da quantidade inicial amebas azuis. As azuis e vermelhas restantes podem ser transformadas em vermelhas? Sabemos que uma azul e uma vermelha pode ser transformada em três vermelhas. Então, como temos azuis, façamos a seguinte transformação: azuis e vermelhas em vermelhas.

Assim, além das amebas azuis, teremos vermelhas (que foram transformadas) junto com outras desta mesma cor (que não foram usadas na última transformação), totalizando .

(b) Encontremos uma invariante neste problema. Aqui mostraremos que a quantidade de vermelhas somado com o dobro da quantidade de azuis nunca muda com as transformações. Analisemos cada transformação.

  • Duas amebas vermelhas se juntam, se transformam em uma única ameba azul.

Se tivermos inicialmente vermelhas e azuis, após esta transformação teremos e azuis. Em ambos os casos, a soma das quantidades de vermelhas com o dobro da de azuis é igual a .

  • Uma ameba vermelha se junta com uma ameba azul, as duas se transformam em três amebas vermelhas.

Análogo ao caso anterior.

  • Duas amebas azuis se juntam, elas se transformam em quatro amebas vermelhas.

Análogo ao caso anterior.

Na configuração inicial, a soma das quantidades de vermelhas com o dobro da de azuis é , enquanto a configuração do item (b) tem essa quantidade igual a . Logo, é impossível sair de uma e chegar na outra.

Exemplo (Cone Sul 1993)[]

Em uma mesa está uma pilha com discos que de forma incremental deve ser convertida em pilhas com três discos cada. Cada passo consiste em selecionar uma pilha removendo um dos seus discos. E então a pilha que sobrou é separada em duas pilhas. Existe uma sequência de passos que pode realizar este processo?

(a)

(b)

Solução: Considere e o números discos e o de pilhas, no começo, respectivamente. Vamos entender o que cada movimento faz com o número de pilhas e de discos.

O número de discos diminui em unidade (pois tiramos um disco) e o número de pilhas aumenta unidade (pois transformamos uma pilha em duas). Desta forma, a soma das quantidades de pilhas e discos é sempre a mesma (encontramos nossa invariante). Esta soma deve ser a mesma do começo, ou seja, .

(a) Na primeira rodada, e . Logo, .

Vamos pensar, agora, na última rodada. Nela haverão apenas pilhas com três discos, ou seja, o número de discos deve ser o triplo do número de pilhas. Assim, a soma do número de pilhas com o de discos de ser um múltiplo de .

Mas como essa soma é sempre a mesma, segue que, é um múltiplo de . Absurdo. Logo, é impossível realizar este processo.

(b) Análogo.

Exemplo (OBM 2022 - Níveis 2 e 3)[]

Um jogo para uma pessoa tem as seguintes regras: inicialmente há dez pilhas de pedras, com pedras, respectivamente. Uma jogada consiste em fazer uma das duas seguintes operações:

(i) escolher duas pilhas, cada uma com pelo menos duas pedras, juntá-las e depois adicionar mais duas pedras a nova pilha;

(ii) escolher uma pilha com pelo menos 4 pedras, tirar duas pedras dela e separá-la em duas pilhas de quantidades de pedras positivas escolhidas pelo jogador.

O jogo continua até não ser mais possível fazer uma operação. Mostre que o número de pilhas com apenas uma pedra ao final do jogo é sempre o mesmo, independentemente de como se realizam as jogadas.

Observação: A versão do Nível 2 desse problema tem três itens:

(a) Dê um exemplo de uma sequência de jogadas que conduzem ao término do jogo.

(b) Faça uma tabela informando qual é o número total de pedras e o número de pilhas no início e após cada uma das 5 primeiras operações no seu exemplo acima.

E o item (c) é equivalente ao problema do Nível 3. Não resolveremos os primeiros dois itens.

Solução: Sejam e o número de pilhas depois de jogadas; temos, claramente, que e . Os dois tipos de jogadas mudam e das seguintes maneiras:

A jogada (i) transforma em e transforma em .

A jogada (ii) transforma em e transforma em .

Observe que a diferença sempre aumenta ou diminui por 3, o que significa que o resto de numa divisão por 3 é sempre constante. No nosso caso, .

Outra coisa a se observar é que o jogo só acaba quando . De fato, ou são impossíveis pelo que acabamos de provar, e significaria que há pelo menos mais 3 pedras do que pilhas, ou seja, uma das jogadas ainda pode ser feita porque existe ou uma pilha com 4 pedras ou um par pilhas com pelo menos 2 pedras. Assim, como afirmamos, o jogo só acaba quando ou, de modo equivalente, . Aqui, todas as pilhas no final têm uma pedra.

Seja, então, . Inicialmente, . A jogada (i) transforma em e a jogada (ii) transforma em . Isso significa que as jogadas (i) e (ii) se anulam em relação ao valor de , e também que um aumento no valor de significa mais jogadas (i) do que (ii), e vice-versa.

Suponha, por fim, que conseguimos fazer um jogo completo com jogadas e levamos a . Como o valor de diminuiu, fizemos mais jogadas (ii) do que (i). Quantas a mais? Considerando que fizemos jogadas (ii) a mais, levamos a , ou seja:

Fizemos 15 jogadas (ii) a mais, e isso levou a pilhas de tamanho 1 (porque agora ). De modo equivalente, poderíamos argumentar que levamos a , confirmando que . Ou seja, o número de pilhas de tamanho 1 no final é sempre constante e é 25.

Exemplo (OBM 2018 - Nível 2)[]

Numa lousa estão escritos inicialmente os números . Para quaisquer dois números e na lousa chamamos de a soma de todos os números na lousa com exceção de e . Uma operação permitida é escolher dois números e na lousa, apagá-los e escrever o número . Após realizar essa operação algumas vezes restam na lousa apenas dois números e , com .

a) Quantas operações foram realizadas?

b) Determine o maior valor possível para .

Solução:

a) Cada operação diminui por 1 a quantidade de números na lousa (escrevemos um e apagamos dois), então, se começamos com 10 números e terminamos com 2, 8 operações foram realizadas.

b) Efetuar essa operação uma única vez parece trabalhoso especialmente por causa da parte do , fazê-la 8 vezes é ainda pior. Vamos pegar um caso menor: 1,2,3.

Se fizermos a operação com 1 e 2, apagaremos os dois e ficaremos com os números 3 e

Se fizermos a operação com 1 e 3, apagaremos os dois e ficaremos com os números 2 e

Se fizermos a operação com 2 e 3, apagaremos os dois e ficaremos com os números 1 e

Curioso... parece que sempre sobram os números e . O que 11 tem de especial aqui? Vamos fazer outro caso e ver se ajuda: os números 2,3 e 4.

Se escolhermos o par (2,3) ficam 4 e

Se escolhermos (2,4) ficam 3 e

Finalmente, escolhendo (3,4) ficam 2 e:

Agora, parece que sobram e , o que significa que 11 não era especial em geral nem 26. Vamos ver se conseguimos descobrir, com números genéricos e :

Se fizermos a operação com e , ficarão e:

Ou seja, sempre sobram e . Parece que a soma dos produtos dois a dois dos números é importante de alguma maneira.

Compensa analisar melhor o que essa operação faz, especialmente em casos maiores. Se os números estão na lousa e fazemos a operação com e , ficam os números , e

Aqui, não parece que sobrou algo como vimos no caso de três letras. E se experimentarmos fazer o produto desse resultado com e separadamente? Afinal, como vimos, a soma dos produtos dos números dois a dois é importante. Ficaremos, então, com:

E

Somando isso a cd, teremos:

Ora, ora! Parece que, mesmo depois que fizemos a operação, a soma dos produtos dois a dois dos números não mudou! Ela é a nossa invariante, certo? Ainda não provamos isso. Vamos fazer um procedimento similar para provar o caso geral.

Sejam os números da lousa. Se fizermos a operação com e , ficaremos com e o número

Se calcularmos a soma dos produtos dois a dois dos novos números, ficamos com:

Que é exatamente a soma dos produtos dois a dois dos nossos números iniciais, ou seja, é uma invariante! Agora sim estamos em condições de finalizar o problema.

Como restam apenas e na lousa, deve ser igual à soma dos produtos dois a dois dos números de 1 até 10, que por enquanto chamaremos de . Assim,

Queremos maximizar , o que equivale a minimizar . Como a operação não nos dá números menores que 1 e o menor número na lousa inicial é 1, é o menor valor possível. Donde é o maior valor possível! Podemos calcular à mão, lembrando que :

Assim, o maior valor possível de é .

Exemplo (Cone Sul 2020)[]

Há uma pilha com 15 fichas sobre uma mesa. A cada passo, Pedro escolhe uma das pilhas na mesa com fichas, divide em duas pilhas com e fichas e escreve no quadro o produto . Ele continua até ter 15 pilhas com 1 ficha cada. Determine todos os valores possíveis para a soma final dos números escritos no quadro.

Observação: Esse problema é bem parecido com esse outro da OBM, mas a solução é bem diferente.

Solução: Fazendo casos pequenos, podemos notar que 2 fichas sempre resulta numa soma igual a 2, 3 fichas sempre resulta numa soma igual a 8, 4 fichas numa soma igual a 20 e 5 fichas numa soma igual a 40. Podemos começar a suspeitar que, para um determinado número de fichas, o resultado seja sempre o mesmo. Vamos fazer indução forte para provar isso: já fizemos os casos base, agora vamos supor que para algum , todos os números de fichas menores que fazem Pedro calcular uma soma fixa. Vamos encontrar uma fórmula para essa soma.

Seja a soma escrita com fichas, definindo por convenção que . É possível notar que, se , temos que e e que . Assim, para :

E assim por diante, até concluirmos que . Podemos recordar (ou talvez concluir novamente) que essa soma é igual a . Assim, estamos supondo como hipótese da nossa indução forte que para todo . Vamos provar que .

Vamos supor que repartimos a pilha com fichas em uma de e outra de , de forma que :

Impondo nossa condição da soma e sabendo que e , temos que, pela hipótese:

Reagrupando e expandindo isso:

Como queríamos. Assim, a única soma que Pedro pode obter é .

Exemplo (OBM 2018 - Nível 3 - Adaptada)[]

Azambuja escreve um número racional em uma lousa. Uma operação consiste em apagar e substituí-lo por ; ou por ; ou por se . O objetivo final de Azambuja é escrever o número após realizar uma quantidade finita de operações. Mostre que se o número escrito é , então Azambuja não poderá alcançar o seu objetivo.

Solução: A primeira coisa que incomoda é transformar o número em em , afinal se fizermos a transformação várias vezes, fica um pouco chato de calcular onde iremos parar. Por isso, vale a pena pensar melhor nessa transformar e ver se conseguimos simplificar um pouco. Considere . Seria legal uma expressão mais simples que relacionasse e . Observe que

Seria legal se conseguíssemos fatorar do lado esquerdo da última igualdade. Mas o máximo que conseguimos fazer é . Ainda não dá para terminar. No primeiro parênteses aparece , enquanto no segundo . Vamos forçar então uma boa fatoração. Como podemos fazer esse sumir? Se dividirmos os dois lados de por :

Está quase "fatorável", afinal podemos escrever Para forçarmos no segundo parênteses, vamos subtrair dos dois lados de :

Por isso, vale a pena considerarmos e . Assim . E quanto as transformações que levam em ou ? Observe que essas transformações levam em ou . Por isso, pode ser levado por elas em ou . Ou ainda podemos transformá-lo em tal que .

Nosso problema pode ser reduzido a: começar com (afinal se, e somente se, ) e somar ou ou transformar em de forma que . Já simplificamos. Mas parece que podemos simplificar mais. Notemos que

Se fizermos e , então . Nosso problema se resume às seguintes regras: começamos com (afinal se, e somente se, ) podemos transformar em (afinal se é transformado em , então é transformado em ) ou em (pelo mesmo raciocínio) ou em de forma que .

Agora parece mais fácil controlarmos as transformações. Como começa racional, ele vai ser racional o tempo todo. Tomemos com . Vamos ver o que acontece com essa fração quando fazemos as transformações. Para facilitar a escrita, vamos associar a fração com um par e ver o que ocorre com ele.

Se transformarmos em isso equivale a transformar em (afinal ). Pelo mesmo raciocínio, levar em equivale a levar em . Finalmente, transformar em de forma que equivale a transformar em (de fato, ).

Observe que começarmos em (ou seja, com o par ) e Azambuja quer parar em (com efeito, por e , e por isso, se, e somente se, ), ou seja, em .

Será que existe algum invariante nas transformações? Observe que se é formado por dois ímpares, então , e também são. Desta forma, os dois números sempre serão ímpares, independente das transformações. Logo, é impossível chegar em .

Invariantes e Restos[]

Prestar atenção em restos pode ser uma estratégia interessante para encontrarmos invariantes.

Algo importante aqui é o seguinte: somar um múltiplo de a um número não altera o seu resto na divisão por .

Exemplo (IMO 1996)[]

São dados um inteiro positivo e um tabuleiro retangular com dimensões , . O retângulo é dividido em uma grade de quadrados unitários. Os seguintes movimentos são permitidos no tabuleiros: pode-se mover de um quadrado para outro apenas se a distância entre os centros dos dois quadrados é . A tarefa é encontrar uma sequência de movimentos conduzindo do quadrado com como vértice ao quadrado que possui como vértice.

(a) Mostre que a tarefa não pode ser feita se é divisível por ou .

(b) Prove que a tarefa é possível se .

(c) A tarefa pode ser feita quando ?

Solução:

(a) Para ajudar o nosso raciocínio, vamos colocar coordenadas. Considere o conjunto de todas as coordenadas com e inteiros tais que e . Queremos ir de a .

Vale a pena estudar melhor como são os movimentos. Observe que cada movimento equivale a andar unidades na horizontal e na vertical. Por isso, cada movimento transforma em . Mas o enunciado nos diz que só podemos fazer o movimento se a distância entre os pontos for . E agora, podemos calcular a distância entre e ? Sim ela é

.

Você pode saber um pouco mais sobre isso aqui. Precisamos que

Agora sim podemos analisar cada caso separadamente.

(i) é par

Aqui e possuem a mesma paridade, de onde segue que e também possuem. Conseguimos usar isso para encontrar alguma invariante? Sim: a paridade da soma das coordenadas não muda com os movimentos. De fato, ela passa de para e paridade de um número não muda se somarmos um número par (no caso é par, pois e possuem mesma paridade).

Logo é impossível neste caso ir de a já que a soma das coordenadas do primeiro é par e a do segundo é ímpar.

(ii) é múltiplo de

Aqui também vale que . Podemos dizer algo sobre a divisibilidade de ou por ? Sim: podemos mostrar que e são ambos múltiplos de . Suponha um deles não seja. Sem perda de generalidade, podemos supor que não é múltiplo de . Desta forma, e assim . Além disso, . Por isso, . Contradição. Desta forma, e devem ser múltiplos de .

Por isso, em cada movimento, estamos somando às coordenadas múltiplos de e assim os seus restos na divisão por não se altera. Desta forma, não podemos sair de e chegar em .

(b) Se , então . Vamos procurar as soluções inteiras positivas dessa equação (depois conseguimos achar as soluções inteiras negativas já que é solução se, e somente se, também é). Repare que

Podemos testar . As únicas soluções que iremos encontrar aqui são e . Portanto, as possíveis transformações para a coordenada são e . Uma possível solução usando essas transformações é:

IMO1996Q1

(c) Aqui . Vamos procurar as soluções inteiras positivas desta equação

Ao testarmos encontraremos as soluções e . Assim os possíveis movimentos são aquele que transformam em ou .

Se fizermos alguns movimentos, podemos perceber que as peças ficam em certas regiões específicas. Considere o conjunto das coordenadas tais que e as coordenadas de que não estão em . Onde estão os pontos depois de cada umas das transformações?

Vamos começar analisando os movimentos que transformam em . Ao somarmos ou subtrairmos de um número menor ou igual a e menor ou igual a , iremos obter um número fora desse intervalo. Desta forma, essa transformação leva pontos de em . E quanto aos pontos de ? Será que eles vão para ? Repare que se um ponto está em , então (e nesse caso só podemos somar às coordenadas de levando o ponto para ) ou (aqui só podemos subtrair de levando o ponto para também). Ou seja, esse tipo de transformação leva pontos de para .

Já os movimentos da forma e só podem ser aplicados em pontos de (levando os pontos de para ).

Para sabermos onde um ponto de (que no caso é o ponto inicial ) vai parar após vários movimentos, devemos saber a paridade da quantidade de movimentos que levam em (já que após uma quantidade par deles, paramos em e após uma quantidade ímpar deles, paramos em ). Como esse movimento muda a paridade de e devemos sair de e chegar a , esse tipo de movimento será aplicado uma quantidade ímpar de vezes.

Mas isso fará com que cheguemos em e não faz parte desse conjunto. Logo não podemos cumprir as condições do enunciado para .

Exemplo (IMO 1993)[]

Em um tabuleiro de xadrez infinito, um jogo é jogado conforme as regras a seguir. No começo, peças são colocadas no tabuleiro em um bloco por de quadrados adjacentes, uma peça em cada quadrado. Um movimento no jogo é um pulo na direção horizontal ou na vertical sobre um quadrado adjacente ocupado para um quadrado desocupado imediatamente depois. A peça que foi pulada é removida.

Encontre todos os valores de para os quais o jogo pode terminar com apenas uma peça sobrando no tabuleiro.

Solução: Ao invés de mexer com casas de uma vez, vale a pena começarmos com casos pequenos para ver se conseguimos alguma ideia interessante. No caso não conseguimos fazer movimento, afinal precisamos de duas pedras para fazer um movimento. Desta forma, neste caso, irá sobrar somente uma peça e esse valor de satisfaz as condições do enunciado.

E quanto a ? Existe uma sequência de movimentos possível:

IMO1993q3

Após mexer um pouco no caso , ele pode parecer impossível. Mas existe um movimento interessante que podemos perceber no meio do processo:

IMO1993q31

Olha só que legal: sempre que tivermos três peças em um tabuleiro e uma peça do lado, podemos "limpar" esse tabuleiro e voltar a peça para a posição original. Observe que isso também vale para um tabuleiro . Será que isso nos ajuda a resolver o problema para o caso ?

IMO1993Q32

Vamos dividir o tabuleiro em cinco subtabuleiros e conforme a figura anterior. Será que conseguimos "limpar" todos esses cinco subtabuleiros? Para facilitar nossa escrita, vamos chamar a peça do quadradinho da linha e coluna de . Podemos

  • usar a peça para limpar o 1° subtabuleiro;
  • depois usar a peça para limpar o 2º subtabuleiro;
  • usar a peça para limpar o 3º subtabuleiro;
  • usar a peça para limpar o 4° subtabuleiro;
  • usar a peça para limpar o 5° subtabuleiro.

Após isso teremos apenas a peça , ou seja, o caso satisfaz as condições do enunciado. Será que essa ideia nos ajuda em casos mais gerais? Sim: para , podemos reduzir um quadrado para um usando a mesma ideia.

IMO1993Q33

Desta forma, se tivermos um tabuleiro , podemos reduzi-lo para um e assim por diante até chegar em um . Ou seja, todo que possui resto na divisão por satisfaz as condições do enunciado. De modo análogo, podemos mostrar que o mesmo vale para todo que possui resto na divisão por .

Resta verificarmos o que acontece com os números que são múltiplos de . Suponha, por absurdo, que podemos terminar o jogo em um tabuleiro . Sejam , e o número de peças nos quadrados tais que deixe restos , e na divisão por . Vale a pena entendermos quanto essas quantidades valem inicialmente e o que ocorre com elas em cada movimento até encontrar uma invariante.

Vamos calcular . Observe que deixa resto na divisão por nos seguintes casos:

  • e deixam resto ;
  • deixa resto e deixa resto ;
  • deixa resto e deixa resto .

Agora precisamos saber quantos números deixam restos , e na divisão por . Os números que deixam resto na divisão por são , ou seja, existem deles. Já também existem números que deixam resto . Eles são . Finalmente os números que deixam resto na divisão por são .

Com isso, as peças tais que deixa resto na divisão por são

Desta maneira, . Analogamente, . E ocorre com esses valores em cada movimento? Observe como é cada tipo de movimento:

IMO1993Q34substituta

Vamos preencher os quadrados com os números (soma da linha com a coluna). Em qualquer um dos quatro movimentos possíveis estaremos mexendo com três valores consecutivos para (e assim os seus restos serão, em alguma ordem, , e ). Observe que dois deles somem e um deles aparece. Por isso, dois dos números , e diminuem uma unidade e o outro irá aumentar uma unidade.

Mais ainda, cada movimento muda a paridade de cada um dos números , e . Mas observe que esses números começam com a mesma paridade. Depois de cada movimento, eles continuarão tendo a mesma paridade. Mas, para sobrar uma peça, deveríamos ter no final um dos valores igual a e os outros dois iguais a zero. Isso não pode acontecer. Logo não podemos terminar com uma peça quando é múltiplo de . Finalmente, o jogo termina se, e somente se, é múltiplo de .

Colorações[]

Colorir objetos da figura te ajuda a separar. É como se você separasse os objetos para diferenciá-los.

Exemplo (Cone Sul 2000)[]

No plano cartesiano, considere os pontos de coordenadas inteiras. Uma operação consiste em:

Escolher um destes pontos e realizar uma rotação de no sentido anti-horário, com centro neste ponto.

É possível, através de uma sequência dessas operações, levar o triângulo de vértices e no triângulo de vértices e ?

Solução: Considere duas cores e . Pinte de e depois pinte todos os outros pontos de coordenadas inteiras com as cores e , alternadamente, conforme a figura a seguir.

ConeSul2000q6

Vamos entender melhor como foi feita a pintura. Pintamos um ponto com a cor se, e somente se, e possuem a mesma paridade (isto é, é par), enquanto será pintado de se, e somente se, e possuem paridades diferentes (isto é, é ímpar).

Mostremos que a cor de um ponto é invariante de fazermos a operação descrita no enunciado. A imagem da rotação de no sentido anti-horário de em torno de será . Vejamos que e possuem mesma paridade, para qualquer que seja .

A soma das coordenadas de é enquanto a soma das de é . Estes dois números possuem a mesma paridade, pois sua soma é par. Com efeito, .

Desta forma, a rotação não altera a cor de um ponto. Assim, as cores do vértices do triângulo não alteram com a rotação. Observe que o primeiro triângulo possui um ponto da cor e dois da cor , enquanto o segundo possui dois da cor e um da cor . Com isso é impossível transformarmos o primeiro triângulo no segundo.

Exemplo (Cone Sul 2009)[]

Ana e Beto jogam em um tabuleiro de linhas e colunas. Primeiro Ana divide o tabuleiro em zonas. Cada zona é formada por casas adjacentes alinhadas vertical ou horizontalmente, como mostra a figura.

ConeSul2009q4

Depois, Beto escreve em cada casa um dos números , de modo que a soma dos números de cada zona seja igual a . Beto ganha se a soma dos números escritos em cada uma das colunas do tabuleiro é um número primo; caso contrário, Ana ganha. Demonstre que Beto tem uma estratégia vencedora.

Solução: Vamos colorir o tabuleiro com três cores e conforme a figura a seguir.

Aqui aparece a nossa invariante: não importa como Ana coloca as peças, cada uma terá uma casa da cor , uma da cor e uma da cor . Como Beto agora pode escrever os números de tal forma que em cada zona a soma é ? Basta que ele escreva nas casas de cores e e nas casas de cor . Assim a soma dos números de cada zona será .

Enquanto a soma das colunas? Observe que em todas elas, duas das cores aparecem vezes, enquanto uma delas aparece vezes. Se a cor que aparece vezes for ou , então a soma dos números desta coluna será . Já se a cor que aparece vezes for , então a soma dos números desta coluna será . Logo, Beto possui a estratégia vencedora.

Exemplo (Cone Sul 2014)[]

Em uma lousa, estão escritos os números inteiros de a , inclusive. A operação válida é escolher dois números e , apagar eles e no seu lugar escrever o mínimo múltiplo comum do par e o máximo divisor comum do par .

Mostre que, sem importar a quantidade de operações que sejam realizadas, a soma dos números que ficam escritos na lousa é sempre maior que .

Solução: (É necessário saber Desigualdades para ler esta solução) Como

,

segue que o produto dos números do quadro é invariante.

Sejam os números após uma sequência qualquer de operações. Pela Desigualdade das Médias,

Monovariantes[]

Algo é chamado de monovariante se nunca aumenta ou nunca diminui.

Exemplo (Cone Sul 2011)[]

Em uma lousa estão escritos os números inteiros positivos de até inclusive. Em cada momento, Pedro apaga dois números da lousa, e , e escreve o número . Pedro repete o procedimento até que sobre apenas um número. Demonstrar que este número será menor que , sem importar quais números Pedro escolha em cada momento.

Solução: (É necessário saber Desigualdades para ler esta solução) Existe um monovariante aqui: a soma dos inversos nunca diminui. Para isto basta provarmos que

A estratégia aqui é escrever de modo equivalente até que a gente tenha algo que saiba que é verdadeiro. A desigualdade anterior equivale a

Como esta desigualdade é verdade, segue que também o é. Se provarmos que, no começo, a soma dos inversos é maior do que e isso nunca diminui, logo, no final, a soma continuará sendo maior do que . Em outras palavras, devemos provar que

A estratégia aqui será dividir em várias partes aqui. Observe que

Se somarmos estas desigualdades e lembrarmos que ,

Função Potencial[]

É uma função que associamos a alguma configuração do problema. Devemos encontrar alguma que nos dê uma invariância ou monovariância. Para isto, podemos forçar o resultado.

Exemplo (OBM 2001 - 3ª Fase - Nível 3)[]

Temos uma fileira longa de copos e pedras no copo central (copo ). Os seguintes movimentos são permitidos:

Movimento do tipo

OBM2001q6n31

Se há pelo menos uma pedra no copo e pelo menos uma no copo podemos fazer uma pedra que está no copo pular para o copo eliminando uma pedra do copo .

Movimento do tipo

OBM2001q6n32

Se há pelo menos duas pedras no copo podemos pular uma para o copo e outra para o copo .

Demonstre o seguinte fato: fazendo os seguintes movimentos do tipo ou durante um tempo suficientemente longo sempre chegaremos a uma configuração a partir da qual não é mais possível fazer nenhum desses dois tipos de movimento. Além disso essa configuração final não depende da escolha de movimentos durante o processo.

Solução: Comecemos provando a primeira parte do enunciado: que não é possível fazer nenhuma jogada depois de um certo tempo. Vamos atribuir um tipo de função potencial que faça as energias diminuírem nos dois tipos de movimento. Para isto, tomaremos uma energia associada ao sistema da seguinte forma

onde é a posição da pedra e é uma valor positivo que iremos determinar que faz as energias diminuírem. Vamos entender o que acontece com a energia quando ocorre o movimento . Nele, sumirão as bolas das posições e e aparecerá uma na posição .

OBM2001q6n33

Se a energia após o movimento for e a antes dele for , então

Esta energia diminui se, e somente se,

Se dividirmos ambos os lados por ,

Vejamos o que acontece quando fazemos um movimento .

OBM2001q6n34

Duas bolas na posição sumirão e aparecerão uma na posição e outra na posição . Assim a energia nova será

A energia aqui diminui se, e somente se,

Se dividirmos ambos os lados por ,

Basta tomarmos que satisfaça e . Por exemplo . Aliás, qualquer tal que . Para este valor, a energia sempre diminui, ou seja, ela será um monovariante.

Vejamos que as pedras não podem ir tão à esquerda assim. Considere uma posição tal que , onde é a energia inicial. Como a energia, a cada movimento, sempre diminui, segue que para quaisquer movimentos que se faça, as pedras nunca ficarão em alguma posição menor ou igual a (de fato, lembre-se que se , então ). Ou seja, existe uma "barreira à esquerda".

Vamos usar isto para provar por indução no número de pedras que é impossível realizar uma sequência infinita de movimentos. Comecemos com o caso . Como precisamos de pelo menos duas pedras para fazer algum dos movimentos, com é impossível fazer algum movimento, ou seja, é impossível fazer uma quantidade infinita de movimentos.

Façamos agora o passo indutivo. Consideremos pedras. Sabemos que a posição da pedra mais à esquerda nunca aumenta. De fato, nenhum dos movimento ou fará com que a pedra mais a esquerda esteja agora mais à direita. Como existe uma "barreira à esquerda", a partir de um certo ponto, a pedra à esquerda também não será mais movimentada e só restará as outras pedras. Por hipótese de indução, é impossível realizar infinitos movimentos com elas. Isto prova a primeira parte do problema.

Vamos agora provar a segunda parte do problema, ou seja, que a configuração inicial não depende da escolha de movimentos durante o processo. Para isto, vamos supor, por absurdo, que existam duas configurações e distintas (entenderemos aqui as configurações como conjuntos de pedras). No final, provaremos que elas são iguais.

Já sabemos que dada uma configuração, independente dos movimentos, existirá algum momento em que é impossível se mover. Vamos chamar isto de configuração parada.

Vejamos se conseguimos fazer uma energia que seja invariante a cada processo. Consideraremos a energia do copo e a energia total do sistema que é dada pela soma das energias de cada copo.

Lembre-se que no movimento , sumirão as bolas das posições e e aparecerá uma na posição . Assim, a energia nova será

A energia irá continuar a mesma se, e somente se,

Já no movimento , duas bolas na posição sumirão e aparecerão uma na posição e outra na posição . Com isso,

A energia será invariante se, e somente se,

Quais valores devemos escolher para que satisfaçam e ? Observe que parece uma sequência de Fibonacci "ao contrário", já que e . Como faremos para reajustar isto?

Tomemos a posição mais à direita das configurações e e . Tomaremos . Vejamos que esta escolha irá satisfazer e . Comecemos com a primeira. Se combinarmos a definição de com a definição da sequência de Fibonacci,

E quanto a ? Se usarmos a definição de e a definição da sequência de Fibonacci primeiro para e depois para ,

Desta forma, esta energia é invariante. Consideremos as energias e das posições finais e . Como esta energia é a mesma do começo, segue que . Vamos mostrar que . Para fazermos isto, usaremos um algoritmo. Ele consistirá no seguinte: tomaremos e as pedras mais à esquerda de e , respectivamente. Depois mostraremos que . Daí bastará repetir o processo para e . O processo deverá ser feito até terminar.

Algoritmo:

(1) Sejam e as pedras mais à esquerda de e , respectivamente. Provaremos que . Vamos supor, por absurdo, que eles não possuem posições iguais. Podemos dizer, sem perda de generalidade, que . Provaremos que, neste caso, . Comecemos observando que

Mostraremos que o maior valor que pode ter é menor do que . Mas qual o valor máximo de ? Observemos algumas características de para descobrir.

Como é uma configuração parada, não podemos ter duas pedras em posições consecutivas (de fato, se tivéssemos, poderíamos fazer o movimento ). Além disso, não podemos ter duas pedras no mesmo copo (pois se isto ocorresse, daria para fazer o movimento ).

Ou seja, para a energia ser máxima, devemos ter copos preenchidos de dois em dois. E isto vai depender da paridade de e . Se e tiverem a mesma paridade, então será máxima quando todos os copos das posições tiverem pedras. Assim

Por uma propriedade da sequência de Fibonacci, esta última expressão é igual a

Já se e tiverem paridades diferentes, então será máxima quando todos os copos das posições tiverem pedras.Assim

Por uma propriedade da sequência de Fibonacci, esta última expressão é igual a

Desta forma, se usarmos o que concluimos em ambos os casos, além de e que a configuração possui pelo menos a pedra ,

Absurdo. Logo .

(2) Volte para o passo 1, porém ao invés de considerar e , considere e .

O algoritmo deve ser repetido até as configurações acabarem. Provaremos daí que e são a mesma configuração, ou seja, a configuração final não depende da escolha de movimentos.

Lugares Para Estudar[]

Vídeos[]

Bibliografia[]

  • D. Djukic, V. Jankovic, I. Matic, N. Petrovic : The IMO Compendium 1959-2009, Springer, 2011.
Advertisement